0 Daumen
408 Aufrufe

367AC41F-841D-46A4-AEF8-A5F752ECE4E2.jpeg

Text erkannt:

Aufgabe 2
Es bezeichne \( D \) das rechtwinklige Dreieck in \( \mathbb{R}^{2} \) mit den Eckpunkten \( (0,0),(1,0) \) und \( (1,1) \). Weiter sei
\( \mathbf{1}_{D}: \mathbb{R}^{2} \rightarrow \mathbb{R}, \quad(x, y) \mapsto \mathbf{1}_{D}(x, y):=\left\{\begin{array}{ll} 1, & (x, y) \in D \\ 0, & (x, y) \notin D \end{array}\right. \)
die Indikatorfunktion auf \( D \).
(a) Bestimmen Sie den unteren beziehungsweise oberen Jordan-Inhalt der Menge \( D \). Verwenden Sie dafür die Zerlegungen
\( Z_{n}:=\left\{\left(\frac{i}{n}, \frac{j}{n}\right): i, j \in\{0,1, \ldots, n\}\right\}, \quad n \in \mathbb{N} . \)

Aufgabe: Hier soll der untere und obere Jordan Inhalt mit Hilfe einer Folge von Zerlegungen berechnet werden.


Problem/Ansatz: In der vorliegenden Lösung verstehe ich die rot markierten Zeilen nicht:

1.Problem:Warum ist die Funktion konstant 0,wenn j >i+1 usw.

2.Problem: Wie kommt man auf den Indexshift bei der Summation

Vielen Dank im Voraus.323A2F37-0EA8-4912-BFA6-FBB4C7B3AAA3.jpeg

Text erkannt:

\( \sum \limits_{i=1}^{k} i=\frac{k(k+1)}{2}, \quad k \in \mathbb{N} . \)
(a) Es sei \( n \in \mathbb{N} \). Wir definieren für \( i, j \in\{1, \ldots, n\} \)
\( Q_{i, j}:=\left[\frac{i-1}{n}, \frac{i}{n}\right] \times\left[\frac{j-1}{n}, \frac{j}{n}\right] . \)
Damit gilt \( \left|Q_{i, j}\right|=\frac{1}{n^{2}} \) für alle \( i, j \in\{1, \ldots, n\} \). Ist \( j>i+1 \), so ist die Funktion \( \mathbf{1}_{D} \) auf dem Quadrat \( Q_{i, j} \) konstant 0 . Im Falle \( j<i \) ist \( \mathbf{1}_{D} \) auf \( Q_{i, j} \) konstant 1 . Gilt hingegen \( j=i \) oder \( j=i+1 \), so erhalten wir
Insgesamt erhalten wir für die Untersumme
und analog für die Obersumme
\( \begin{aligned} O\left(\mathbf{1}_{D}, Z_{n}\right) &=\sum \limits_{i=1}^{n} \sum \limits_{j=1}^{n} \sup _{D}\left(Q_{i, j}\right) \cdot\left|Q_{i, j}\right|=\frac{1}{n^{2}}\left(n-1+\sum \limits_{i=1}^{n} \sum \limits_{j=1}^{i} 1\right)=\frac{1}{n^{2}}\left(n-1+\sum \limits_{i=1}^{n} i\right) \\ &=\frac{1}{n^{2}}\left(n-1+\frac{n(n+1)}{2}\right)=\frac{n^{2}+3 n-2}{2 n^{2}} \end{aligned} \)
Damit folgen
\( \begin{array}{l} \bar{J}(D)=\bar{J}\left(\mathbf{1}_{D},[0,1]^{2}\right) \leq \lim \limits_{n \rightarrow \infty} O\left(\mathbf{1}_{D}, Z_{n}\right)=\lim \limits_{n \rightarrow \infty} \frac{n^{2}+3 n-2}{2 n^{2}}=\frac{1}{2}, \\ \underline{J}(D)=\underline{J}\left(\mathbf{1}_{D},[0,1]^{2}\right) \geq \lim \limits_{n \rightarrow \infty} U\left(\mathbf{1}_{D}, Z_{n}\right)=\lim \limits_{n \rightarrow \infty} \frac{n^{2}-n}{2 n^{2}}=\frac{1}{2} . \end{array} \)
Somit erhalten wir \( \frac{1}{2} \geq \bar{J}\left(\mathbf{1}_{D},[0,1]^{2}\right) \geq \underline{J}\left(\mathbf{1}_{D},[0,1]^{2}\right) \geq \frac{1}{2} \) und schlieBlich
\( \bar{J}\left(\mathbf{1}_{D},[0,1]^{2}\right)=\underline{J}\left(\mathbf{1}_{D},[0,1]^{2}\right)=\frac{1}{2} \)

Avatar von

Da wird von der charakteristischen Funktion auf \( D \) gesprochen. Was ist \( D \)?

Ich hab die Aufgabenstellung jetzt hinzugefügt,hab ich vergessen.D sei ein rechtwinkliges Dreieck im R^2.Die Eckpunkte des Dreiecks D sind (0,0),(1,1),(1,0).

1 Antwort

0 Daumen

Das Quadrat \( Q_{i,j} \) liegt im Dreieck \( D \) wenn die Oberkante des Quadarats unterhalb oder auf der Winkelhalbierenden liegt. D.h. es muss gelten \( \frac{j}{n} \le \frac{i-1}{n} \) oder anders geschrieben \( j \le i-1 \) oder auch \( j < i \)

In diesem Fall ist \( \mathbb{1}_D\left(Q_{i,j}\right) = 1 \)

Das Quadrat \( Q_{i,j} \) liegt nicht im Dreieck \( D \) wenn die Unterkante des Quadarats oberhalb der Winkelhalbierenden liegt. D.h. es muss gelten \( \frac{j-1}{n} > \frac{i}{n} \) oder anders geschrieben \( j > i+1 \)

In diesem Fall ist \( \mathbb{1}_D\left(Q_{i,j}\right) = 0 \)

Damit bleiben noch zwei Möglichkeiten offen, nämlich \( j = i+1 \) und \( j=i \)

Bei \( j = i \) teilt die Winkelhalbierende gerade das Quadrat \( Q_{i,j} = Q_{i,i} \) Deshalb ist das

\( \inf \mathbb{1}_D\left(Q_{i,j}\right) = 0 \) und das \( \sup \mathbb{1}_D\left(Q_{i,j}\right) =  1 \) weil Werte des Quadrats innerhalb und außerhalb des Dreiecks \( D \) liegen.

Im Fall \( j = i+1 \) liegt die untere rechte Kante des Quadrats gerade auf der Winkelhalbierenden. Deshalb gibt es auch hier Werte des Quadrats, die im und nicht im Dreieck \( D \) liegen und es gilt wieder

\( \inf \mathbb{1}_D\left(Q_{i,j}\right) = 0 \) und das \( \sup \mathbb{1}_D\left(Q_{i,j}\right) =  1 \)


Jetzt zur Berechnung der Untersumme

$$ U(\mathbb{1}_D,Z_n ) = \sum_{i=1}^n \sum_{j=1}^n \inf \mathbb{1}_D\left(Q_{i,j}\right) \cdot \left| Q_{i,j} \right| $$

Wegen \( \left| Q_{i,j} \right| = \frac{1}{n^2} \) gilt also

$$ U(\mathbb{1}_D,Z_n ) = \frac{1}{n^2} \sum_{i=1}^n \sum_{j=1}^n \inf \mathbb{1}_D\left(Q_{i,j}\right) $$

Wie aber oben beschrieben ist das Infimum für \( j > i \) aber Null und für \( j \le i-1 \) Eins. Deshalb hat die Summierung nur bis \( j = i-1 \) zu erfolgen.

Es gilt also

$$ U(\mathbb{1}_D,Z_n ) = \frac{1}{n^2} \sum_{i=1}^n \sum_{j=1}^{i-1} 1 $$ da \( i-1 < 1 \) für \( i = 1 \) gilt, beginnt die Summierung bzgl. des Indexes \( i \) erst bei \( 2 \), also

$$ U(\mathbb{1}_D,Z_n ) = \frac{1}{n^2} \sum_{i=2}^n \sum_{j=1}^{i-1} 1 $$

Bei der Obersumme geht es in etwa ebenso.

Ich habe es so berechnet

$$ O(\mathbb{1}_D,Z_n ) = \frac{1}{n^2} \sum_{i=1}^n \sum_{j=1}^{i+1} 1 = \frac{1}{n^2} \left( \frac{n(n+1)}{2} + n \right) $$

mein Ergebnis unterscheidet sich etwas von Deiner Lösung. Entweder ist die Musterlösung falsch oder ich hab mich verrechnet. Am Endergebnis ändert sich allerdings nichts.

Avatar von 39 k

Ein anderes Problem?

Stell deine Frage

Willkommen bei der Mathelounge! Stell deine Frage einfach und kostenlos

x
Made by a lovely community